Math, asked by preetika15, 1 year ago

If X³ - 2x²y²+5x + y = 5,then dy/dx at x =1 y =1 is dx (a) 4/3 (b)-5/4 (c) 4/5 (d) -4/3​

Answers

Answered by MaheswariS
10

\textbf{Given:}

\mathsf{x^3-2x^2y^2+5x+y=5}

\textbf{To find:}

\mathsf{\dfrac{dy}{dx}\;at\;x=1,y=1}

\textbf{Solution:}

\textsf{Consider,}

\mathsf{x^3-2x^2y^2+5x+y=5}

\textsf{Differentiate with respect to 'x'}

\mathsf{3x^2-2(x^2\,2y\dfrac{dy}{dx}+y^2\,2x)+5+\dfrac{dy}{dx}=0}

\mathsf{3x^2-4\,x^2y\dfrac{dy}{dx}-4y^2\,x+5+\dfrac{dy}{dx}=0}

\mathsf{-4\,x^2y\dfrac{dy}{dx}+\dfrac{dy}{dx}=-3x^2+4\,y^2\,x-5}

\mathsf{(1-4\,x^2y)\dfrac{dy}{dx}=-3x^2+4\,y^2\,x-5}

\mathsf{\dfrac{dy}{dx}=\dfrac{-3x^2+4\,y^2\,x-5}{1-4\,x^2y}}

\mathsf{Now,}

\mathsf{\left(\dfrac{dy}{dx}\right)_{(1,1)}}

\mathsf{=\dfrac{-3+4-5}{1-4}}

\mathsf{=\dfrac{-4}{-3}}

\mathsf{=\dfrac{4}{-3}}

\implies\boxed{\mathsf{\left(\dfrac{dy}{dx}\right)_{(1,1)}=\dfrac{4}{3}}}

\textbf{Answer:}

\textsf{Option (a) is correct}

Answered by rahul1347u
2

Answer:

f(x, y) = x³ + 2x²y² + 3x² + 3y²

fxy is

A)

Similar questions